Download as pdf or txt
Download as pdf or txt
You are on page 1of 3

EE 590 B (PMP): In-Class Activity #3

Solutions

Wednesday, November 2, 2016

Tamara Bonaci
Department of Electrical Engineering
University of Washington, Seattle

Problem 1. System Properties


Consider the system:
y(t) = x(4t + 1)
Is the given system:
• Linear?
• Time-invariant?
• Causal?
• Memoryless?

Solution:
Linearity: The given system is linear. To see that, we can separately look at the superposition
property as follows:

T {αx1 (t) + βx2 (t)} = α x1 (4t + 1) +β x2 (4t + 1) = αy1 (t) + βy2 (t) (1)
| {z } | {z }
y1 (t) y2 (t)

Time-invariance: The given system is not time invariant. To see that let’s see what happens when
we time shift the output, vs. time-shifting the input:

y(t − t0 ) = x(4(t − t0 ) + 1) (2)

For time-shifted input z(t) = x(t − t0 ), we have:

y(t) = z(4t + 1) = x(4t − t0 + 1) (3)

Since the given equations are not equal, the given system is not time invariant.
Causality: The given system is not causal. To see that, let’s consider the system output at time
t = 0:
y(t = 0) = x(4 · 0 + 1) = x(1) (4)
Obviously, the given system depends on the future value of the input signal, so the system is not causal.
Memorylessness: The given system is not memoryless. To see that, let’s consider the system
output at time t = −1:
y(t = −1) = x(4 · −1 + 1) = x(−3) (5)
Obviously, the given system depends on the past values of the input signal, so it is not memoryless.
Problem 2. Fourier Series
For the continuous-time periodic signal:
   
2π 5π
x(t) = 2 + cos t + 4 sin t
3 3

1
determine the fundamental frequency ω0 and the Fourier series coefficients ak such that:

X
x(t) = ak ejkω0 t
k=−∞

Solution:
To find the fundamental frequency of the given sum of periodic signals, let’s consider individual com-
ponents:

ω01 =
3

ω02 =
3
So, if follows that the fundamental frequency of the sum equals ω0 = π3 .
Now, to find the coefficients of the Fourier series, let’s use Euler’s formulas as follows:

ej2π/3 + e−j2π/3
 
2π 1 2π 1 2π
cos = = ej 3 + e−j 3
3 2 2 2
ej5π/3 − e−j5π/3
 
5π 5π 5π
4 sin = 2 = −2jej 3 + 2je−j 3
3 2j

From the upper equations, it follows that the Fourier series coefficients equal:
1
a0 = 2, a2 = a−2 = , a5 = −2j, a−5 = 2j
2

Problem 3. Fourier Transform I


Consider the signal x(t), defined as:

x(t) = u(t + 2) − u(t − 1).

Please find the Fourier transform X(jω) of the given signal.

Solution:
By using the Fourier Transform pair for x(t) = rect Tt  X(jω) = 2 sin(ωT /2) ωT
 
ω = T sinc 2 , and
transform property:
F T {x(t − t0 )} = e−jt0 ω X(jω)
we have:
2 sin( 23 ω)
    
1 3ω
FT x t+ = ejω/2 F T {rect(t/3)} = ejω/2 = 3ejω/2 sinc (6)
2 ω 2

Problem 4. Fourier Transform II


Using the analysis and synthesis integrals of Fourier transforms, find X(jω) if the signal x(t) is given
as:
X∞
x(t) = αk δ(t − kT )
k=0

and |α| < 1.

2
Solution:
Z ∞ ∞
X
X(jω) = αk δ(t − kT )e−jωt dt
−∞ k=0

X Z ∞
= k
α δ(t − kT )e−jωt dt
k=0 −∞

X∞ ∞
X
k −jωkT
= α e = (αe−jωT )k
k=0 k=0

Since |αe−jωT | = |α||e−jωT | = |α||1| = |α| < 1, using formula for geometric series:

X a
ark =
1−r
k=0

we get that the Fourier transform of the given signal equals:


1
X(jω) =
1 − αe−jωT

You might also like